LSAT and Law School Admissions Forum

Get expert LSAT preparation and law school admissions advice from PowerScore Test Preparation.

 acp25
  • Posts: 13
  • Joined: Sep 21, 2017
|
#49892
I am struggling to figure out why #19's right answer is A. Please help and thank you in advance.
 Malila Robinson
PowerScore Staff
  • PowerScore Staff
  • Posts: 296
  • Joined: Feb 01, 2018
|
#49986
Hi acp25,
The question is asking for the main conclusion of Maria's argument. Her argument is:
"Calling any state totalitarian is misleading: it implies total state control of all aspects of life. The real world contains no political entity exercising literally total control over even one such aspect. This is because any system of control is inefficient, and, therefore, its degree of control is partial."

Her conclusion is: "Calling any state totalitarian is misleading"

The answer that restates that most closely is A: "No state can be called totalitarian without inviting a mistaken belief."

Hope that helps!
-Malila
 grunerlokka
  • Posts: 22
  • Joined: Jul 07, 2020
|
#77119
Why is Answer B wrong? B says: "To be totalitarian, a state must totally control society." Is this answer not after all a paraphrase of Maria's main point?

Is it that the conditional in answer B is misleading? Or is it that the answer misses the "calling/called" qualifier, which answer A has? If the problem is the latter (i.e the fact that "calling/called" is missing from B) is this really that consequential in differentiating wrong/right answer, with respect to future questions I might encounter of this same form? Thanks!
 Frank Peter
PowerScore Staff
  • PowerScore Staff
  • Posts: 99
  • Joined: May 14, 2020
|
#77130
Hi Gruner,

I would say that the point that (B) raises is operating more as a premise - the main conclusion here is that calling any state totalitarian is misleading. Why? Because a totalitarian state must totally control society, which no totalitarian state has managed to do. (B) doesn't really capture the aspect of Maria's argument that one might be misled by referring to a state as totalitarian.
User avatar
 alexis.la
  • Posts: 13
  • Joined: Jul 14, 2021
|
#88730
Hey! I had an easy time selecting an answer with this question because I was confident I had the right conclusion, but I was wrong.
Maria says: "This is because any system of control is inefficient, and, therefore, its degree of control is partial".
"therefore" immediately made me think "ok this is the conclusion", so I selected answer C: "the degree of control exercised by a state is necessarily partial", because this seemed to be a paraphrase of what I thought was the conclusion.

I know I can always use the Conclusion Identification Method, but I can't afford to use that on every question because of time constraints. Do you have any tips on 'not falling' for indicator words? In this case, 'therefore' wasn't actually followed by the conclusion, rather the conclusion was the first sentence. But I was trying to be mechanistic.
Thanks in advance.
 Robert Carroll
PowerScore Staff
  • PowerScore Staff
  • Posts: 1787
  • Joined: Dec 06, 2013
|
#88769
Alexis,

Consider that we have another indicator also: "because". So Maria is saying "The second sentence is true because..." So you have all the evidence that the second sentence is also some kind of conclusion, with the third sentence providing some evidence for it. So, if the issue is mechanically looking at indicators, "This is because..." tells you that you have a premise that's evidence for something earlier as well, so you should be on guard that there may be multiple conclusions here, and you need to be careful to identify which is the main conclusion.

Robert Carroll

Get the most out of your LSAT Prep Plus subscription.

Analyze and track your performance with our Testing and Analytics Package.